Verbal questions from any Manhattan Prep GMAT Computer Adaptive Test. Topic subject should be the first few words of your question.
priyanka.bishnoi
Students
 
Posts: 2
Joined: Mon Jun 21, 2010 6:43 am
 

Manhattan CAT 2 Question

by priyanka.bishnoi Tue Jul 06, 2010 10:40 am

Because of a rare type of fungus that killed off many cacao trees in Brazil, there was an unusually meager harvest of cacao beans this year. The wholesale price of cocoa solids and cocoa butter has increased significantly and is unlikely to fall in the foreseeable future. As a result, the retail price of chocolate is certain to increase within six months.

The answer to which of the following questions would provide information relevant to evaluating the argument above?


A)Has the price of cocoa solids and cocoa butter remained steady during other periods of poor harvest?
B)Are consumers willing to spend more for chocolate?
C)Have the prices of other ingredients in chocolate decreased recently?
D)What percentage of cacao trees in Brazil were affected by the fungus?
E)Can the fungus be eliminated within the next six months?

I am not convinced with the explanation for this question.
I chose E.Because if we assume "Yes" as an answer to this then the prices of cocoa products will get controlled. And if we assume the answer to this question as "No" then the chocolate price will increase.

The answer is C but i am not convinced with this. This is because if we consider the the answer to this question as "yes" then we don't know is the decrease sufficient enough to compensate the increase in the price of chocolate.And if assume the answer to this question as "yes" then surely the price of chocolate will not fall.

Is my approach to such questions wrong? Please guide.
tim
Course Students
 
Posts: 5665
Joined: Tue Sep 11, 2007 9:08 am
Location: Southwest Airlines, seat 21C
 

Re: Manhattan CAT 2 Question

by tim Mon Aug 02, 2010 5:08 pm

priyanka.bishnoi Wrote:I am not convinced with the explanation for this question.
I chose E.Because if we assume "Yes" as an answer to this then the prices of cocoa products will get controlled. And if we assume the answer to this question as "No" then the chocolate price will increase.


The issue is not whether the price will "get controlled". Eliminating the fungus in a couple of months won't necessarily keep the prices of retail chocolate from increasing (remember to refrain from bringing any outside knowledge you may have about the sensitivity of the chocolate futures market).. :)

And no, assuming E is answered in the negative absolutely does not lead inevitably to an increase in the chocolate price, precisely because C provides an alternative scenario..

priyanka.bishnoi Wrote:The answer is C but i am not convinced with this. This is because if we consider the the answer to this question as "yes" then we don't know is the decrease sufficient enough to compensate the increase in the price of chocolate.And if assume the answer to this question as "yes" then surely the price of chocolate will not fall.


Of course we don't know that the decrease is sufficient. However, all we need to know is that having this information will be relevant in helping us determine whether the decrease is sufficient. Remember to look at the wording of the problem!
Tim Sanders
Manhattan GMAT Instructor

Follow this link for some important tips to get the most out of your forum experience:
https://www.manhattanprep.com/gmat/forums/a-few-tips-t31405.html
itsmeaakash3
Forum Guests
 
Posts: 12
Joined: Sun Jun 24, 2012 2:40 am
 

Re: Manhattan CAT 2 Question

by itsmeaakash3 Thu Jun 06, 2013 11:09 pm

I don't agree with C.

Lets assume a chocolate costs 5$ and it has three ingredients with following prices :
Ingredient A : .1 $
Ingredient B : ,1 $
Ingredient Chocolate : 4.8 $

Assuming prices of a & b reduce to .5$ each and chocolate price increases by 1.5 $ then obviously price is bound to increase.
jlucero
Forum Guests
 
Posts: 1102
Joined: Wed May 12, 2010 1:33 am
 

Re: Manhattan CAT 2 Question

by jlucero Fri Jun 21, 2013 1:17 pm

itsmeaakash3 Wrote:I don't agree with C.

Lets assume a chocolate costs 5$ and it has three ingredients with following prices :
Ingredient A : .1 $
Ingredient B : ,1 $
Ingredient Chocolate : 4.8 $

Assuming prices of a & b reduce to .5$ each and chocolate price increases by 1.5 $ then obviously price is bound to increase.


Take a look at the question being asked:

The answer to which of the following questions would provide information relevant to evaluating the argument above?

It doesn't say that this guarantees the price won't increase. It just asks us what else we would want to know. Yes, there are certain scenarios where the prices of other products go down and the price of chocolate goes up, but there are also scenarios where the price of chocolate would be unaffected. This is common on the GMAT- as you look for a correct answer choice, don't expect it to complete the argument: it's just another piece of the puzzle that helps make the scenario clearer.
Joe Lucero
Manhattan GMAT Instructor
itsmeaakash3
Forum Guests
 
Posts: 12
Joined: Sun Jun 24, 2012 2:40 am
 

Re: Manhattan CAT 2 Question

by itsmeaakash3 Thu Jul 18, 2013 3:24 pm

How about B ?

What if people don't want to buy expensive ones ?
jlucero
Forum Guests
 
Posts: 1102
Joined: Wed May 12, 2010 1:33 am
 

Re: Manhattan CAT 2 Question

by jlucero Fri Aug 16, 2013 3:49 pm

itsmeaakash3 Wrote:How about B ?

What if people don't want to buy expensive ones ?


I'm not going to overthrow the economic model of supply and demand here, but I will say that this principle usually takes place over time. If the costs go up, the retail price of chocolate will likely go up at some point in the next 6 months. Notice that the conclusion is not that it will STAY up, but rather that it will "increase within six months". If consumers don't want to buy the more expensive chocolate, than maybe that price will come down with time.
Joe Lucero
Manhattan GMAT Instructor
RohitM174
Forum Guests
 
Posts: 1
Joined: Wed Apr 30, 2014 11:32 pm
 

Re: Manhattan CAT 2 Question

by RohitM174 Fri May 16, 2014 11:13 am

jlucero Wrote:
itsmeaakash3 Wrote:How about B ?

What if people don't want to buy expensive ones ?


I'm not going to overthrow the economic model of supply and demand here, but I will say that this principle usually takes place over time. If the costs go up, the retail price of chocolate will likely go up at some point in the next 6 months. Notice that the conclusion is not that it will STAY up, but rather that it will "increase within six months". If consumers don't want to buy the more expensive chocolate, than maybe that price will come down with time.


Thanks for the information, Point Taken.

However if we check Option A, it also gives us similar reason to believe that prices did not rise in other incidence despite of Cocoa shortage. I see it equally emphasizing the point vis a vis Option C, Since option C as well is providing us another possibility without any exact or sure information.

How can we we rule out A in this case and should be take away in such cases.

Seeking your help

Regards,
RonPurewal
Students
 
Posts: 19744
Joined: Tue Aug 14, 2007 8:23 am
 

Re: Manhattan CAT 2 Question

by RonPurewal Mon May 19, 2014 6:26 pm

RohitM174 Wrote:However if we check Option A, it also gives us similar reason to believe that prices did not rise in other incidence despite of Cocoa shortage.


Those prices have increased this time. That's a fact"”in the passage.
Even if the same thing didn't happen on previous occasions, we don't care, because it happened this time.

(The passage jumps to the price of chocolate without any good justification, so you can reasonably anticipate that the issue will have something to do with that jump.)
JbhB682
Course Students
 
Posts: 520
Joined: Fri May 16, 2014 2:13 pm
 

Re: Manhattan CAT 2 Question

by JbhB682 Wed Feb 17, 2021 1:00 pm

Hello I thought C was pointless for this question because even though C may have an IMPACT on the retail price, it has nothing to do with the premises tying to the conclusion specifically

Let me give an example of what I mean

JbhB682 Wrote:Source : Mprep Blog [https://www.manhattanprep.com/gmat/blog/tackling-a-gmatprep-cr-evaluate-problem/]

In order to increase its profits, MillCo plans to reduce costs by laying off any non-essential employees

option 1) Whether revenues will be affected adversely enough to threaten MillCo’s profit structure.
option 2) Whether MillCo might reduce its costs by eliminating any contract workers.

Here option 2 is considered wrong even though reducing contract workers will impact profits, option 2 is considered irrelevant

My thinking was : because option 2 has nothing to do with the premise tying itself to the conclusion (the premise is talking about non-essential workers and the impact of getting rid of non-essential workers specifically on the profit structure)

Option 1 is better because if revenues are impacted (As a result of the decision to lay off non essential workers) - that would be important.


With this same thinking, option C is like the option 2 in my example - it seems irrelevant even though the price of other ingredients does impact retail prices.

Sure, price of OTHER ingredients will impact retail price BUT price of other ingredients has nothing to do with the premise tying itself to the conclusion specifically [ the prices of other ingredients changing has no relationship to the rare type of fungus or the unusual meager harvest]

Thoughts ?
Last edited by JbhB682 on Wed Feb 17, 2021 1:16 pm, edited 3 times in total.
JbhB682
Course Students
 
Posts: 520
Joined: Fri May 16, 2014 2:13 pm
 

Re: Manhattan CAT 2 Question

by JbhB682 Wed Feb 17, 2021 1:03 pm

^^^ last thought

Option C would make sense if the price of other ingredients decreased recently AS A RESULT of the fungus or meager harvest --- then option C would make more sense

Without the additional underline in option C -- i don't think option C is a clear winner as prices of other ingredients could have decreased recently for some other reason completely unrelated to the fungus or the meager harvest , thereby making option C irrelevant like option 2 in my example.
esledge
Forum Guests
 
Posts: 1181
Joined: Tue Mar 01, 2005 6:33 am
Location: St. Louis, MO
 

Re: Manhattan CAT 2 Question

by esledge Sat Feb 20, 2021 4:11 pm

JbhB682 Wrote:^^^ last thought

Option C would make sense if the price of other ingredients decreased recently AS A RESULT of the fungus or meager harvest --- then option C would make more sense

Without the additional underline in option C -- i don't think option C is a clear winner as prices of other ingredients could have decreased recently for some other reason completely unrelated to the fungus or the meager harvest , thereby making option C irrelevant like option 2 in my example.
I think you might be placing too much emphasis on evaluating a choice that ties the premise and conclusion together. The way this (cacao) question is worded, you only need something relevant to evaluating the argument (i.e. the outcome or conclusion). Thus, the right answer must affect retail price...and that's it. The prices of other ingredients are relevant in calculating the retail price of chocolate, so (C) is the right answer.

You might say "What about the as a result part of the conclusion?" Well, that's the question writer tying the fungus & meager harvest to the predicted increase in the retail price of chocolate. In other words, you already know from the givens that the cocoa solids and cocoa butter have already increased in cost as a result of the fungus. But to evaluate whether the price of the finished product (chocolate) will increase, you just care about whether the other ingredients cost more or less, and you don't have to care about why their prices may have changed.
JbhB682 Wrote:Source : Mprep Blog [https://www.manhattanprep.com/gmat/blog/tackling-a-gmatprep-cr-evaluate-problem/]

In order to increase its profits, MillCo plans to reduce costs by laying off any non-essential employees

option 1) Whether revenues will be affected adversely enough to threaten MillCo’s profit structure.
option 2) Whether MillCo might reduce its costs by eliminating any contract workers.

Here option 2 is considered wrong even though reducing contract workers will impact profits, option 2 is considered irrelevant


This one is slightly different, as the "how" of the plan is integral to the conclusion: Millco plans to increase profits by reducing costs by eliminating employee group A.

Option 2 is off topic because "Can Millco reduce costs by eliminating employee group B?" doesn't address the efficacy of Millco's plan to increase profits by eliminating employee group A.

It's like if I said "I am going to improve my work-at-home output by taking a break every hour."
option 1: Whether my breaks will be so long that the lost work time threatens any efficiency gained by taking the break.
option 2: Whether I might improve my work-at-home output by investing in better lighting and a more comfortable chair.

Option 2 addresses the real-life concern, but it doesn't evaluate the argument as presented.
Emily Sledge
Instructor
ManhattanGMAT